Show that $lim_n sum_{k=1}^nfrac{B_k}{k!},frac{n^underline{k-1}}{n^{k-1}}=sum_{k=1}^inftyfrac{B_k}{k!}$











up vote
3
down vote

favorite












I need to show that



$$lim_{ntoinfty} sum_{k=1}^nfrac{B_k}{k!},frac{n^underline{k-1}}{n^{k-1}}=sum_{k=1}^inftyfrac{B_k}{k!}$$



where $n^underline{k-1}:=prod_{j=0}^{k-2}(n-j)$ is a falling factorial and the $B_k$ are the Bernoulli numbers, and I know that the RHS of above converges to $1/(e-1)$. I had two attempts:



1) First I tried to use the dominated convergence theorem setting $a_n(k):=frac{B_k}{k!},frac{n^underline{k-1}}{n^{k-1}}chi_{[1,n]}(k)$, then clearly $lim_n a_n(k)=B_k/k!$ for each $kinBbb N_{ge 1}$, however I dont know if $sum_{k=1}^infty|B_k|/k!$ converges, and I dont know any absolutely convergent series that dominates, so Im stuck at this step.



2) A more elementary approach



$$left|sum_{k=1}^infty a_n(k)-sum_{k=1}^inftyfrac{B_k}{k!}right|leleft|sum_{k=1}^M(a_n(k)-B_k/k!)right|+sum_{k=M+1}^nleft|1-frac{n^underline{k-1}}{n^{k-1}}right|+left|sum_{k=n+1}^inftyfrac{B_k}{k!}right|$$



such that $|B_k/k!|<1$ for $kge M+1$. Then taking limits above we have that



$$lim_{ntoinfty}left|sum_{k=1}^inftyleft(a_n(k)-frac{B_k}{k!}right)right|lelim_{ntoinfty}sum_{k=M+1}^nleft(1-frac{n^underline{k-1}}{n^{k-1}}right)$$



for any fixed enough large $M$. Then if I can show that for each $epsilon>0$ there is some $MinBbb N$ such that



$$lim_{ntoinfty}sum_{k=M+1}^nleft(1-frac{n^underline{k-1}}{n^{k-1}}right)<epsilon$$



then Im done. However it is not clear how to accomplish (or if it is possible) this task. I thought about use the Stirling approximation on $n^underline{k-1}/n^{k-1}$, however it is not clear that I can apply an asymptotic expression inside a series, so Im again stuck.





There is some easy way (the more elementary the better) to show the converge of the limit of the title? Thank you.










share|cite|improve this question
























  • Since $n^{underline{k-1}} =0$ if $k-2 ge n$ the series $sum_{k=M+1}^infty (1-n^{underline{k-1}}/n^k)$ is divergent!
    – p4sch
    Nov 19 at 14:05












  • @p4sch I fixed it, thank you. However I find a solution!
    – Masacroso
    Nov 19 at 14:09






  • 1




    The same problem occurs in the last line: $M^{underline{k-1}} =0$ if $k ge M+2$. You would like to show that $sum_{k=M+1}^n (1-n^{underline{k-1}}/n^k)$ can be made small for all large $n ge N$ independent of $n$.
    – p4sch
    Nov 19 at 14:31










  • There is also another problem in your post: Note that $n^{underline{k-1}}$ has only $k-1$ terms in the product. Thus $n^{underline{k-1}}/n^k = n^{-1} prod_{j=0}^{k-2} (1-j/n) le n^{-1}$. I.e. your limes is zero, but $ 0 > (2-e)/(e-1) =1/(e-1)-1 = sum_{k=1}^infty B_k/k!$.
    – p4sch
    Nov 19 at 14:43










  • @p4sch yes, this was a persistent typographic error, thank you
    – Masacroso
    Nov 19 at 14:47















up vote
3
down vote

favorite












I need to show that



$$lim_{ntoinfty} sum_{k=1}^nfrac{B_k}{k!},frac{n^underline{k-1}}{n^{k-1}}=sum_{k=1}^inftyfrac{B_k}{k!}$$



where $n^underline{k-1}:=prod_{j=0}^{k-2}(n-j)$ is a falling factorial and the $B_k$ are the Bernoulli numbers, and I know that the RHS of above converges to $1/(e-1)$. I had two attempts:



1) First I tried to use the dominated convergence theorem setting $a_n(k):=frac{B_k}{k!},frac{n^underline{k-1}}{n^{k-1}}chi_{[1,n]}(k)$, then clearly $lim_n a_n(k)=B_k/k!$ for each $kinBbb N_{ge 1}$, however I dont know if $sum_{k=1}^infty|B_k|/k!$ converges, and I dont know any absolutely convergent series that dominates, so Im stuck at this step.



2) A more elementary approach



$$left|sum_{k=1}^infty a_n(k)-sum_{k=1}^inftyfrac{B_k}{k!}right|leleft|sum_{k=1}^M(a_n(k)-B_k/k!)right|+sum_{k=M+1}^nleft|1-frac{n^underline{k-1}}{n^{k-1}}right|+left|sum_{k=n+1}^inftyfrac{B_k}{k!}right|$$



such that $|B_k/k!|<1$ for $kge M+1$. Then taking limits above we have that



$$lim_{ntoinfty}left|sum_{k=1}^inftyleft(a_n(k)-frac{B_k}{k!}right)right|lelim_{ntoinfty}sum_{k=M+1}^nleft(1-frac{n^underline{k-1}}{n^{k-1}}right)$$



for any fixed enough large $M$. Then if I can show that for each $epsilon>0$ there is some $MinBbb N$ such that



$$lim_{ntoinfty}sum_{k=M+1}^nleft(1-frac{n^underline{k-1}}{n^{k-1}}right)<epsilon$$



then Im done. However it is not clear how to accomplish (or if it is possible) this task. I thought about use the Stirling approximation on $n^underline{k-1}/n^{k-1}$, however it is not clear that I can apply an asymptotic expression inside a series, so Im again stuck.





There is some easy way (the more elementary the better) to show the converge of the limit of the title? Thank you.










share|cite|improve this question
























  • Since $n^{underline{k-1}} =0$ if $k-2 ge n$ the series $sum_{k=M+1}^infty (1-n^{underline{k-1}}/n^k)$ is divergent!
    – p4sch
    Nov 19 at 14:05












  • @p4sch I fixed it, thank you. However I find a solution!
    – Masacroso
    Nov 19 at 14:09






  • 1




    The same problem occurs in the last line: $M^{underline{k-1}} =0$ if $k ge M+2$. You would like to show that $sum_{k=M+1}^n (1-n^{underline{k-1}}/n^k)$ can be made small for all large $n ge N$ independent of $n$.
    – p4sch
    Nov 19 at 14:31










  • There is also another problem in your post: Note that $n^{underline{k-1}}$ has only $k-1$ terms in the product. Thus $n^{underline{k-1}}/n^k = n^{-1} prod_{j=0}^{k-2} (1-j/n) le n^{-1}$. I.e. your limes is zero, but $ 0 > (2-e)/(e-1) =1/(e-1)-1 = sum_{k=1}^infty B_k/k!$.
    – p4sch
    Nov 19 at 14:43










  • @p4sch yes, this was a persistent typographic error, thank you
    – Masacroso
    Nov 19 at 14:47













up vote
3
down vote

favorite









up vote
3
down vote

favorite











I need to show that



$$lim_{ntoinfty} sum_{k=1}^nfrac{B_k}{k!},frac{n^underline{k-1}}{n^{k-1}}=sum_{k=1}^inftyfrac{B_k}{k!}$$



where $n^underline{k-1}:=prod_{j=0}^{k-2}(n-j)$ is a falling factorial and the $B_k$ are the Bernoulli numbers, and I know that the RHS of above converges to $1/(e-1)$. I had two attempts:



1) First I tried to use the dominated convergence theorem setting $a_n(k):=frac{B_k}{k!},frac{n^underline{k-1}}{n^{k-1}}chi_{[1,n]}(k)$, then clearly $lim_n a_n(k)=B_k/k!$ for each $kinBbb N_{ge 1}$, however I dont know if $sum_{k=1}^infty|B_k|/k!$ converges, and I dont know any absolutely convergent series that dominates, so Im stuck at this step.



2) A more elementary approach



$$left|sum_{k=1}^infty a_n(k)-sum_{k=1}^inftyfrac{B_k}{k!}right|leleft|sum_{k=1}^M(a_n(k)-B_k/k!)right|+sum_{k=M+1}^nleft|1-frac{n^underline{k-1}}{n^{k-1}}right|+left|sum_{k=n+1}^inftyfrac{B_k}{k!}right|$$



such that $|B_k/k!|<1$ for $kge M+1$. Then taking limits above we have that



$$lim_{ntoinfty}left|sum_{k=1}^inftyleft(a_n(k)-frac{B_k}{k!}right)right|lelim_{ntoinfty}sum_{k=M+1}^nleft(1-frac{n^underline{k-1}}{n^{k-1}}right)$$



for any fixed enough large $M$. Then if I can show that for each $epsilon>0$ there is some $MinBbb N$ such that



$$lim_{ntoinfty}sum_{k=M+1}^nleft(1-frac{n^underline{k-1}}{n^{k-1}}right)<epsilon$$



then Im done. However it is not clear how to accomplish (or if it is possible) this task. I thought about use the Stirling approximation on $n^underline{k-1}/n^{k-1}$, however it is not clear that I can apply an asymptotic expression inside a series, so Im again stuck.





There is some easy way (the more elementary the better) to show the converge of the limit of the title? Thank you.










share|cite|improve this question















I need to show that



$$lim_{ntoinfty} sum_{k=1}^nfrac{B_k}{k!},frac{n^underline{k-1}}{n^{k-1}}=sum_{k=1}^inftyfrac{B_k}{k!}$$



where $n^underline{k-1}:=prod_{j=0}^{k-2}(n-j)$ is a falling factorial and the $B_k$ are the Bernoulli numbers, and I know that the RHS of above converges to $1/(e-1)$. I had two attempts:



1) First I tried to use the dominated convergence theorem setting $a_n(k):=frac{B_k}{k!},frac{n^underline{k-1}}{n^{k-1}}chi_{[1,n]}(k)$, then clearly $lim_n a_n(k)=B_k/k!$ for each $kinBbb N_{ge 1}$, however I dont know if $sum_{k=1}^infty|B_k|/k!$ converges, and I dont know any absolutely convergent series that dominates, so Im stuck at this step.



2) A more elementary approach



$$left|sum_{k=1}^infty a_n(k)-sum_{k=1}^inftyfrac{B_k}{k!}right|leleft|sum_{k=1}^M(a_n(k)-B_k/k!)right|+sum_{k=M+1}^nleft|1-frac{n^underline{k-1}}{n^{k-1}}right|+left|sum_{k=n+1}^inftyfrac{B_k}{k!}right|$$



such that $|B_k/k!|<1$ for $kge M+1$. Then taking limits above we have that



$$lim_{ntoinfty}left|sum_{k=1}^inftyleft(a_n(k)-frac{B_k}{k!}right)right|lelim_{ntoinfty}sum_{k=M+1}^nleft(1-frac{n^underline{k-1}}{n^{k-1}}right)$$



for any fixed enough large $M$. Then if I can show that for each $epsilon>0$ there is some $MinBbb N$ such that



$$lim_{ntoinfty}sum_{k=M+1}^nleft(1-frac{n^underline{k-1}}{n^{k-1}}right)<epsilon$$



then Im done. However it is not clear how to accomplish (or if it is possible) this task. I thought about use the Stirling approximation on $n^underline{k-1}/n^{k-1}$, however it is not clear that I can apply an asymptotic expression inside a series, so Im again stuck.





There is some easy way (the more elementary the better) to show the converge of the limit of the title? Thank you.







real-analysis limits convergence






share|cite|improve this question















share|cite|improve this question













share|cite|improve this question




share|cite|improve this question








edited Nov 19 at 14:51

























asked Nov 19 at 12:16









Masacroso

12.3k41746




12.3k41746












  • Since $n^{underline{k-1}} =0$ if $k-2 ge n$ the series $sum_{k=M+1}^infty (1-n^{underline{k-1}}/n^k)$ is divergent!
    – p4sch
    Nov 19 at 14:05












  • @p4sch I fixed it, thank you. However I find a solution!
    – Masacroso
    Nov 19 at 14:09






  • 1




    The same problem occurs in the last line: $M^{underline{k-1}} =0$ if $k ge M+2$. You would like to show that $sum_{k=M+1}^n (1-n^{underline{k-1}}/n^k)$ can be made small for all large $n ge N$ independent of $n$.
    – p4sch
    Nov 19 at 14:31










  • There is also another problem in your post: Note that $n^{underline{k-1}}$ has only $k-1$ terms in the product. Thus $n^{underline{k-1}}/n^k = n^{-1} prod_{j=0}^{k-2} (1-j/n) le n^{-1}$. I.e. your limes is zero, but $ 0 > (2-e)/(e-1) =1/(e-1)-1 = sum_{k=1}^infty B_k/k!$.
    – p4sch
    Nov 19 at 14:43










  • @p4sch yes, this was a persistent typographic error, thank you
    – Masacroso
    Nov 19 at 14:47


















  • Since $n^{underline{k-1}} =0$ if $k-2 ge n$ the series $sum_{k=M+1}^infty (1-n^{underline{k-1}}/n^k)$ is divergent!
    – p4sch
    Nov 19 at 14:05












  • @p4sch I fixed it, thank you. However I find a solution!
    – Masacroso
    Nov 19 at 14:09






  • 1




    The same problem occurs in the last line: $M^{underline{k-1}} =0$ if $k ge M+2$. You would like to show that $sum_{k=M+1}^n (1-n^{underline{k-1}}/n^k)$ can be made small for all large $n ge N$ independent of $n$.
    – p4sch
    Nov 19 at 14:31










  • There is also another problem in your post: Note that $n^{underline{k-1}}$ has only $k-1$ terms in the product. Thus $n^{underline{k-1}}/n^k = n^{-1} prod_{j=0}^{k-2} (1-j/n) le n^{-1}$. I.e. your limes is zero, but $ 0 > (2-e)/(e-1) =1/(e-1)-1 = sum_{k=1}^infty B_k/k!$.
    – p4sch
    Nov 19 at 14:43










  • @p4sch yes, this was a persistent typographic error, thank you
    – Masacroso
    Nov 19 at 14:47
















Since $n^{underline{k-1}} =0$ if $k-2 ge n$ the series $sum_{k=M+1}^infty (1-n^{underline{k-1}}/n^k)$ is divergent!
– p4sch
Nov 19 at 14:05






Since $n^{underline{k-1}} =0$ if $k-2 ge n$ the series $sum_{k=M+1}^infty (1-n^{underline{k-1}}/n^k)$ is divergent!
– p4sch
Nov 19 at 14:05














@p4sch I fixed it, thank you. However I find a solution!
– Masacroso
Nov 19 at 14:09




@p4sch I fixed it, thank you. However I find a solution!
– Masacroso
Nov 19 at 14:09




1




1




The same problem occurs in the last line: $M^{underline{k-1}} =0$ if $k ge M+2$. You would like to show that $sum_{k=M+1}^n (1-n^{underline{k-1}}/n^k)$ can be made small for all large $n ge N$ independent of $n$.
– p4sch
Nov 19 at 14:31




The same problem occurs in the last line: $M^{underline{k-1}} =0$ if $k ge M+2$. You would like to show that $sum_{k=M+1}^n (1-n^{underline{k-1}}/n^k)$ can be made small for all large $n ge N$ independent of $n$.
– p4sch
Nov 19 at 14:31












There is also another problem in your post: Note that $n^{underline{k-1}}$ has only $k-1$ terms in the product. Thus $n^{underline{k-1}}/n^k = n^{-1} prod_{j=0}^{k-2} (1-j/n) le n^{-1}$. I.e. your limes is zero, but $ 0 > (2-e)/(e-1) =1/(e-1)-1 = sum_{k=1}^infty B_k/k!$.
– p4sch
Nov 19 at 14:43




There is also another problem in your post: Note that $n^{underline{k-1}}$ has only $k-1$ terms in the product. Thus $n^{underline{k-1}}/n^k = n^{-1} prod_{j=0}^{k-2} (1-j/n) le n^{-1}$. I.e. your limes is zero, but $ 0 > (2-e)/(e-1) =1/(e-1)-1 = sum_{k=1}^infty B_k/k!$.
– p4sch
Nov 19 at 14:43












@p4sch yes, this was a persistent typographic error, thank you
– Masacroso
Nov 19 at 14:47




@p4sch yes, this was a persistent typographic error, thank you
– Masacroso
Nov 19 at 14:47










2 Answers
2






active

oldest

votes

















up vote
1
down vote













The Bernoulli numbers are defined
$$frac{z}{e^z-1} = sum_{k=0}^infty frac{B_k}{k!}z^k$$
and this is a holomorphic function on the domain defined by $|mathrm{Im}(z)| < 2pi$. Thus the radius of convergence is $2 pi$. Any power seriers is absolutely convergent in the radius of convergence.



Here, for example we know that $|frac{B_k}{k!} 2^k| leq 1$ for all $k ge N$ and some $N in mathbb{N}$, because the series is convergent for $z=2$. Thus
$$sum_{k=N}^n frac{|B_k|}{k!} = sum_{k=N}^n frac{1}{2^k} le 1. $$
In other words: The series is absolute convergent and this can be used in order to show the claim (by using the dominated convergence theorem).



Second answer: The sum $sum_{k=M+1}^n (1-n^{underline{k-1}}/n^{k-1})$ isn't convergent, i.e. you have you use a better bound for $|B_k|/k!$ in order to get that the series is convergent. We use $1-x ge e^{-x}/2$ for $0 <x le 1/2$ to get
$$tag{1}sum_{k=M+1}^n (1-n^{underline{k-1}}/n^{k-1}) ge frac{1}{2} sum_{kge n/2+2}^n exp(-n^{underline{k-1}}/n^{k-1}),$$
where we used that
$$n^{underline{k-1}}/n^{k-1} = prod_{j=0}^{k-2} (1-j/n) le Big(1-frac{k-2}{n}Big)$$
is at most $1/2$ if $k ge n/2+2$. Now note that $ln(1-x) le -x$ for all $x ge 0$ to obtain also the lower bound
$$ ln (n^{underline{k-1}}/n^{k-1}) = sum_{j=0}^{k-2} ln(1-j/n) le -sum_{j=1}^{k-2} j/n = - frac{(k-1)(k-2)}{2n}.$$
All in all, we see for $n ge 2M-4$ that (1) can be bounded below by
$$frac{1}{2} sum_{k=lfloor n/2+2 rfloor +1}^n exp[-exp {-(k-1)(k-2)/(2n)}].$$
and this is bounded by
$$ frac{n}{2} exp(- exp{-n/8}) rightarrow infty.$$






share|cite|improve this answer



















  • 1




    $|B_k 2^k /k! | le 1$ is nothing else than $|B_k/k!| le 2^{-k}$.
    – p4sch
    Nov 19 at 14:20










  • in your second answer you are saying that $|n^underline{k-1}/n^{k-1}|<1/2$, but this is relative to the value of $k$ and $n$, so you cant make the lower bound on $(1)$ because $n^underline{k-1}/n^{k-1}to 1$ as $ntoinfty$ for fixed $k$
    – Masacroso
    Nov 19 at 21:29












  • $k$ is not fixed, since we sum over $k=M+1$ up to $k=n$. I repaired my answer. You just need to skip sum summands and begin to sum at $k ge n/2+2$.
    – p4sch
    Nov 20 at 8:26




















up vote
0
down vote



accepted










I find a solution using the recursion



$$B_k=-sum_{j=0}^{k-1}binom{k}jfrac{B_j}{k-1+j}tag1$$



and the dominated convergence theorem. From $(1)$ and setting $M:=sup_{kinBbb N}|B_k|/k!$ (what is finite because I know that $sum_{k=1}^infty B_k/k!$ converges) then we have that



$$begin{align}left|frac{B_k}{k!}right|&=left|frac1{k!}sum_{j=0}^{k-1}binom{k}jfrac{B_j}{k-j+1}right|=left|sum_{j=0}^{k-1}frac{B_j}{j!}frac1{(k-j+1)!}right|\
&lefrac{M k}{(k-1)!}sum_{j=0}^{k-1}binom{k-1}j
=frac{Mk}{(k-1)!}2^{k-1}\&=2kMprod_{j=0}^{k-4}frac2{k-1-j}end{align}tag2$$



Then it is easy to check that for $kge 6$ we have that $frac{|B_k|}{k!}le 16Mfrac{k}{(k-3)^3}$, so by the integral test the series $sum_{k=1}^infty |B_k|/k!$ converges.



Now using the first approach on the question we can apply the dominated convergence theorem and find that



$$lim_{ntoinfty}sum_{k=1}^{n+1}frac{B_k}{k!}cdotfrac{n^underline{k-1}}{n^{k-1}}=lim_{ntoinfty}int_{Bbb N_{ge 1}}a_n(k) ,dmathcal H^0=int_{Bbb N_{ge 1}}frac{B_k}{k!},dmathcal H^0=sum_{k=1}^inftyfrac{B_k}{k!}tag3$$



as desired. However I would like to see, if possible, a more simple solution that doesn't use "advanced" theorems as the dominated convergence theorem.



P.S.: $dmathcal H^0$ is just the counting measure.





Essentially the same question appear in a mathematical magazine. In short it says that $n^underline{k-1}in n^{k-1}+O(n^{k-2})$ as $ntoinfty$, thus $frac{n^underline{k-1}}{n^{k-1}}in 1+O(n^{-1})$ and we find that



$$sum_{k=1}^{n+1}frac{B_k}{k!}cdotfrac{n^underline{k-1}}{n^{k-1}}in sum_{k=1}^{n+1}frac{B_k}{k!}+O(n^{-1})sum_{k=1}^{n+1}frac{B_k}{k!}tag4$$



Then in the article the author take limits on $(4)$, "giving" the wanted identity. However this procedure seems wrong as its said in the last link. I added this to the answer for the curious reader.






share|cite|improve this answer























    Your Answer





    StackExchange.ifUsing("editor", function () {
    return StackExchange.using("mathjaxEditing", function () {
    StackExchange.MarkdownEditor.creationCallbacks.add(function (editor, postfix) {
    StackExchange.mathjaxEditing.prepareWmdForMathJax(editor, postfix, [["$", "$"], ["\\(","\\)"]]);
    });
    });
    }, "mathjax-editing");

    StackExchange.ready(function() {
    var channelOptions = {
    tags: "".split(" "),
    id: "69"
    };
    initTagRenderer("".split(" "), "".split(" "), channelOptions);

    StackExchange.using("externalEditor", function() {
    // Have to fire editor after snippets, if snippets enabled
    if (StackExchange.settings.snippets.snippetsEnabled) {
    StackExchange.using("snippets", function() {
    createEditor();
    });
    }
    else {
    createEditor();
    }
    });

    function createEditor() {
    StackExchange.prepareEditor({
    heartbeatType: 'answer',
    convertImagesToLinks: true,
    noModals: true,
    showLowRepImageUploadWarning: true,
    reputationToPostImages: 10,
    bindNavPrevention: true,
    postfix: "",
    imageUploader: {
    brandingHtml: "Powered by u003ca class="icon-imgur-white" href="https://imgur.com/"u003eu003c/au003e",
    contentPolicyHtml: "User contributions licensed under u003ca href="https://creativecommons.org/licenses/by-sa/3.0/"u003ecc by-sa 3.0 with attribution requiredu003c/au003e u003ca href="https://stackoverflow.com/legal/content-policy"u003e(content policy)u003c/au003e",
    allowUrls: true
    },
    noCode: true, onDemand: true,
    discardSelector: ".discard-answer"
    ,immediatelyShowMarkdownHelp:true
    });


    }
    });














    draft saved

    draft discarded


















    StackExchange.ready(
    function () {
    StackExchange.openid.initPostLogin('.new-post-login', 'https%3a%2f%2fmath.stackexchange.com%2fquestions%2f3004849%2fshow-that-lim-n-sum-k-1n-fracb-kk-fracn-underlinek-1nk-1%23new-answer', 'question_page');
    }
    );

    Post as a guest















    Required, but never shown

























    2 Answers
    2






    active

    oldest

    votes








    2 Answers
    2






    active

    oldest

    votes









    active

    oldest

    votes






    active

    oldest

    votes








    up vote
    1
    down vote













    The Bernoulli numbers are defined
    $$frac{z}{e^z-1} = sum_{k=0}^infty frac{B_k}{k!}z^k$$
    and this is a holomorphic function on the domain defined by $|mathrm{Im}(z)| < 2pi$. Thus the radius of convergence is $2 pi$. Any power seriers is absolutely convergent in the radius of convergence.



    Here, for example we know that $|frac{B_k}{k!} 2^k| leq 1$ for all $k ge N$ and some $N in mathbb{N}$, because the series is convergent for $z=2$. Thus
    $$sum_{k=N}^n frac{|B_k|}{k!} = sum_{k=N}^n frac{1}{2^k} le 1. $$
    In other words: The series is absolute convergent and this can be used in order to show the claim (by using the dominated convergence theorem).



    Second answer: The sum $sum_{k=M+1}^n (1-n^{underline{k-1}}/n^{k-1})$ isn't convergent, i.e. you have you use a better bound for $|B_k|/k!$ in order to get that the series is convergent. We use $1-x ge e^{-x}/2$ for $0 <x le 1/2$ to get
    $$tag{1}sum_{k=M+1}^n (1-n^{underline{k-1}}/n^{k-1}) ge frac{1}{2} sum_{kge n/2+2}^n exp(-n^{underline{k-1}}/n^{k-1}),$$
    where we used that
    $$n^{underline{k-1}}/n^{k-1} = prod_{j=0}^{k-2} (1-j/n) le Big(1-frac{k-2}{n}Big)$$
    is at most $1/2$ if $k ge n/2+2$. Now note that $ln(1-x) le -x$ for all $x ge 0$ to obtain also the lower bound
    $$ ln (n^{underline{k-1}}/n^{k-1}) = sum_{j=0}^{k-2} ln(1-j/n) le -sum_{j=1}^{k-2} j/n = - frac{(k-1)(k-2)}{2n}.$$
    All in all, we see for $n ge 2M-4$ that (1) can be bounded below by
    $$frac{1}{2} sum_{k=lfloor n/2+2 rfloor +1}^n exp[-exp {-(k-1)(k-2)/(2n)}].$$
    and this is bounded by
    $$ frac{n}{2} exp(- exp{-n/8}) rightarrow infty.$$






    share|cite|improve this answer



















    • 1




      $|B_k 2^k /k! | le 1$ is nothing else than $|B_k/k!| le 2^{-k}$.
      – p4sch
      Nov 19 at 14:20










    • in your second answer you are saying that $|n^underline{k-1}/n^{k-1}|<1/2$, but this is relative to the value of $k$ and $n$, so you cant make the lower bound on $(1)$ because $n^underline{k-1}/n^{k-1}to 1$ as $ntoinfty$ for fixed $k$
      – Masacroso
      Nov 19 at 21:29












    • $k$ is not fixed, since we sum over $k=M+1$ up to $k=n$. I repaired my answer. You just need to skip sum summands and begin to sum at $k ge n/2+2$.
      – p4sch
      Nov 20 at 8:26

















    up vote
    1
    down vote













    The Bernoulli numbers are defined
    $$frac{z}{e^z-1} = sum_{k=0}^infty frac{B_k}{k!}z^k$$
    and this is a holomorphic function on the domain defined by $|mathrm{Im}(z)| < 2pi$. Thus the radius of convergence is $2 pi$. Any power seriers is absolutely convergent in the radius of convergence.



    Here, for example we know that $|frac{B_k}{k!} 2^k| leq 1$ for all $k ge N$ and some $N in mathbb{N}$, because the series is convergent for $z=2$. Thus
    $$sum_{k=N}^n frac{|B_k|}{k!} = sum_{k=N}^n frac{1}{2^k} le 1. $$
    In other words: The series is absolute convergent and this can be used in order to show the claim (by using the dominated convergence theorem).



    Second answer: The sum $sum_{k=M+1}^n (1-n^{underline{k-1}}/n^{k-1})$ isn't convergent, i.e. you have you use a better bound for $|B_k|/k!$ in order to get that the series is convergent. We use $1-x ge e^{-x}/2$ for $0 <x le 1/2$ to get
    $$tag{1}sum_{k=M+1}^n (1-n^{underline{k-1}}/n^{k-1}) ge frac{1}{2} sum_{kge n/2+2}^n exp(-n^{underline{k-1}}/n^{k-1}),$$
    where we used that
    $$n^{underline{k-1}}/n^{k-1} = prod_{j=0}^{k-2} (1-j/n) le Big(1-frac{k-2}{n}Big)$$
    is at most $1/2$ if $k ge n/2+2$. Now note that $ln(1-x) le -x$ for all $x ge 0$ to obtain also the lower bound
    $$ ln (n^{underline{k-1}}/n^{k-1}) = sum_{j=0}^{k-2} ln(1-j/n) le -sum_{j=1}^{k-2} j/n = - frac{(k-1)(k-2)}{2n}.$$
    All in all, we see for $n ge 2M-4$ that (1) can be bounded below by
    $$frac{1}{2} sum_{k=lfloor n/2+2 rfloor +1}^n exp[-exp {-(k-1)(k-2)/(2n)}].$$
    and this is bounded by
    $$ frac{n}{2} exp(- exp{-n/8}) rightarrow infty.$$






    share|cite|improve this answer



















    • 1




      $|B_k 2^k /k! | le 1$ is nothing else than $|B_k/k!| le 2^{-k}$.
      – p4sch
      Nov 19 at 14:20










    • in your second answer you are saying that $|n^underline{k-1}/n^{k-1}|<1/2$, but this is relative to the value of $k$ and $n$, so you cant make the lower bound on $(1)$ because $n^underline{k-1}/n^{k-1}to 1$ as $ntoinfty$ for fixed $k$
      – Masacroso
      Nov 19 at 21:29












    • $k$ is not fixed, since we sum over $k=M+1$ up to $k=n$. I repaired my answer. You just need to skip sum summands and begin to sum at $k ge n/2+2$.
      – p4sch
      Nov 20 at 8:26















    up vote
    1
    down vote










    up vote
    1
    down vote









    The Bernoulli numbers are defined
    $$frac{z}{e^z-1} = sum_{k=0}^infty frac{B_k}{k!}z^k$$
    and this is a holomorphic function on the domain defined by $|mathrm{Im}(z)| < 2pi$. Thus the radius of convergence is $2 pi$. Any power seriers is absolutely convergent in the radius of convergence.



    Here, for example we know that $|frac{B_k}{k!} 2^k| leq 1$ for all $k ge N$ and some $N in mathbb{N}$, because the series is convergent for $z=2$. Thus
    $$sum_{k=N}^n frac{|B_k|}{k!} = sum_{k=N}^n frac{1}{2^k} le 1. $$
    In other words: The series is absolute convergent and this can be used in order to show the claim (by using the dominated convergence theorem).



    Second answer: The sum $sum_{k=M+1}^n (1-n^{underline{k-1}}/n^{k-1})$ isn't convergent, i.e. you have you use a better bound for $|B_k|/k!$ in order to get that the series is convergent. We use $1-x ge e^{-x}/2$ for $0 <x le 1/2$ to get
    $$tag{1}sum_{k=M+1}^n (1-n^{underline{k-1}}/n^{k-1}) ge frac{1}{2} sum_{kge n/2+2}^n exp(-n^{underline{k-1}}/n^{k-1}),$$
    where we used that
    $$n^{underline{k-1}}/n^{k-1} = prod_{j=0}^{k-2} (1-j/n) le Big(1-frac{k-2}{n}Big)$$
    is at most $1/2$ if $k ge n/2+2$. Now note that $ln(1-x) le -x$ for all $x ge 0$ to obtain also the lower bound
    $$ ln (n^{underline{k-1}}/n^{k-1}) = sum_{j=0}^{k-2} ln(1-j/n) le -sum_{j=1}^{k-2} j/n = - frac{(k-1)(k-2)}{2n}.$$
    All in all, we see for $n ge 2M-4$ that (1) can be bounded below by
    $$frac{1}{2} sum_{k=lfloor n/2+2 rfloor +1}^n exp[-exp {-(k-1)(k-2)/(2n)}].$$
    and this is bounded by
    $$ frac{n}{2} exp(- exp{-n/8}) rightarrow infty.$$






    share|cite|improve this answer














    The Bernoulli numbers are defined
    $$frac{z}{e^z-1} = sum_{k=0}^infty frac{B_k}{k!}z^k$$
    and this is a holomorphic function on the domain defined by $|mathrm{Im}(z)| < 2pi$. Thus the radius of convergence is $2 pi$. Any power seriers is absolutely convergent in the radius of convergence.



    Here, for example we know that $|frac{B_k}{k!} 2^k| leq 1$ for all $k ge N$ and some $N in mathbb{N}$, because the series is convergent for $z=2$. Thus
    $$sum_{k=N}^n frac{|B_k|}{k!} = sum_{k=N}^n frac{1}{2^k} le 1. $$
    In other words: The series is absolute convergent and this can be used in order to show the claim (by using the dominated convergence theorem).



    Second answer: The sum $sum_{k=M+1}^n (1-n^{underline{k-1}}/n^{k-1})$ isn't convergent, i.e. you have you use a better bound for $|B_k|/k!$ in order to get that the series is convergent. We use $1-x ge e^{-x}/2$ for $0 <x le 1/2$ to get
    $$tag{1}sum_{k=M+1}^n (1-n^{underline{k-1}}/n^{k-1}) ge frac{1}{2} sum_{kge n/2+2}^n exp(-n^{underline{k-1}}/n^{k-1}),$$
    where we used that
    $$n^{underline{k-1}}/n^{k-1} = prod_{j=0}^{k-2} (1-j/n) le Big(1-frac{k-2}{n}Big)$$
    is at most $1/2$ if $k ge n/2+2$. Now note that $ln(1-x) le -x$ for all $x ge 0$ to obtain also the lower bound
    $$ ln (n^{underline{k-1}}/n^{k-1}) = sum_{j=0}^{k-2} ln(1-j/n) le -sum_{j=1}^{k-2} j/n = - frac{(k-1)(k-2)}{2n}.$$
    All in all, we see for $n ge 2M-4$ that (1) can be bounded below by
    $$frac{1}{2} sum_{k=lfloor n/2+2 rfloor +1}^n exp[-exp {-(k-1)(k-2)/(2n)}].$$
    and this is bounded by
    $$ frac{n}{2} exp(- exp{-n/8}) rightarrow infty.$$







    share|cite|improve this answer














    share|cite|improve this answer



    share|cite|improve this answer








    edited Nov 20 at 8:26

























    answered Nov 19 at 14:17









    p4sch

    4,130216




    4,130216








    • 1




      $|B_k 2^k /k! | le 1$ is nothing else than $|B_k/k!| le 2^{-k}$.
      – p4sch
      Nov 19 at 14:20










    • in your second answer you are saying that $|n^underline{k-1}/n^{k-1}|<1/2$, but this is relative to the value of $k$ and $n$, so you cant make the lower bound on $(1)$ because $n^underline{k-1}/n^{k-1}to 1$ as $ntoinfty$ for fixed $k$
      – Masacroso
      Nov 19 at 21:29












    • $k$ is not fixed, since we sum over $k=M+1$ up to $k=n$. I repaired my answer. You just need to skip sum summands and begin to sum at $k ge n/2+2$.
      – p4sch
      Nov 20 at 8:26
















    • 1




      $|B_k 2^k /k! | le 1$ is nothing else than $|B_k/k!| le 2^{-k}$.
      – p4sch
      Nov 19 at 14:20










    • in your second answer you are saying that $|n^underline{k-1}/n^{k-1}|<1/2$, but this is relative to the value of $k$ and $n$, so you cant make the lower bound on $(1)$ because $n^underline{k-1}/n^{k-1}to 1$ as $ntoinfty$ for fixed $k$
      – Masacroso
      Nov 19 at 21:29












    • $k$ is not fixed, since we sum over $k=M+1$ up to $k=n$. I repaired my answer. You just need to skip sum summands and begin to sum at $k ge n/2+2$.
      – p4sch
      Nov 20 at 8:26










    1




    1




    $|B_k 2^k /k! | le 1$ is nothing else than $|B_k/k!| le 2^{-k}$.
    – p4sch
    Nov 19 at 14:20




    $|B_k 2^k /k! | le 1$ is nothing else than $|B_k/k!| le 2^{-k}$.
    – p4sch
    Nov 19 at 14:20












    in your second answer you are saying that $|n^underline{k-1}/n^{k-1}|<1/2$, but this is relative to the value of $k$ and $n$, so you cant make the lower bound on $(1)$ because $n^underline{k-1}/n^{k-1}to 1$ as $ntoinfty$ for fixed $k$
    – Masacroso
    Nov 19 at 21:29






    in your second answer you are saying that $|n^underline{k-1}/n^{k-1}|<1/2$, but this is relative to the value of $k$ and $n$, so you cant make the lower bound on $(1)$ because $n^underline{k-1}/n^{k-1}to 1$ as $ntoinfty$ for fixed $k$
    – Masacroso
    Nov 19 at 21:29














    $k$ is not fixed, since we sum over $k=M+1$ up to $k=n$. I repaired my answer. You just need to skip sum summands and begin to sum at $k ge n/2+2$.
    – p4sch
    Nov 20 at 8:26






    $k$ is not fixed, since we sum over $k=M+1$ up to $k=n$. I repaired my answer. You just need to skip sum summands and begin to sum at $k ge n/2+2$.
    – p4sch
    Nov 20 at 8:26












    up vote
    0
    down vote



    accepted










    I find a solution using the recursion



    $$B_k=-sum_{j=0}^{k-1}binom{k}jfrac{B_j}{k-1+j}tag1$$



    and the dominated convergence theorem. From $(1)$ and setting $M:=sup_{kinBbb N}|B_k|/k!$ (what is finite because I know that $sum_{k=1}^infty B_k/k!$ converges) then we have that



    $$begin{align}left|frac{B_k}{k!}right|&=left|frac1{k!}sum_{j=0}^{k-1}binom{k}jfrac{B_j}{k-j+1}right|=left|sum_{j=0}^{k-1}frac{B_j}{j!}frac1{(k-j+1)!}right|\
    &lefrac{M k}{(k-1)!}sum_{j=0}^{k-1}binom{k-1}j
    =frac{Mk}{(k-1)!}2^{k-1}\&=2kMprod_{j=0}^{k-4}frac2{k-1-j}end{align}tag2$$



    Then it is easy to check that for $kge 6$ we have that $frac{|B_k|}{k!}le 16Mfrac{k}{(k-3)^3}$, so by the integral test the series $sum_{k=1}^infty |B_k|/k!$ converges.



    Now using the first approach on the question we can apply the dominated convergence theorem and find that



    $$lim_{ntoinfty}sum_{k=1}^{n+1}frac{B_k}{k!}cdotfrac{n^underline{k-1}}{n^{k-1}}=lim_{ntoinfty}int_{Bbb N_{ge 1}}a_n(k) ,dmathcal H^0=int_{Bbb N_{ge 1}}frac{B_k}{k!},dmathcal H^0=sum_{k=1}^inftyfrac{B_k}{k!}tag3$$



    as desired. However I would like to see, if possible, a more simple solution that doesn't use "advanced" theorems as the dominated convergence theorem.



    P.S.: $dmathcal H^0$ is just the counting measure.





    Essentially the same question appear in a mathematical magazine. In short it says that $n^underline{k-1}in n^{k-1}+O(n^{k-2})$ as $ntoinfty$, thus $frac{n^underline{k-1}}{n^{k-1}}in 1+O(n^{-1})$ and we find that



    $$sum_{k=1}^{n+1}frac{B_k}{k!}cdotfrac{n^underline{k-1}}{n^{k-1}}in sum_{k=1}^{n+1}frac{B_k}{k!}+O(n^{-1})sum_{k=1}^{n+1}frac{B_k}{k!}tag4$$



    Then in the article the author take limits on $(4)$, "giving" the wanted identity. However this procedure seems wrong as its said in the last link. I added this to the answer for the curious reader.






    share|cite|improve this answer



























      up vote
      0
      down vote



      accepted










      I find a solution using the recursion



      $$B_k=-sum_{j=0}^{k-1}binom{k}jfrac{B_j}{k-1+j}tag1$$



      and the dominated convergence theorem. From $(1)$ and setting $M:=sup_{kinBbb N}|B_k|/k!$ (what is finite because I know that $sum_{k=1}^infty B_k/k!$ converges) then we have that



      $$begin{align}left|frac{B_k}{k!}right|&=left|frac1{k!}sum_{j=0}^{k-1}binom{k}jfrac{B_j}{k-j+1}right|=left|sum_{j=0}^{k-1}frac{B_j}{j!}frac1{(k-j+1)!}right|\
      &lefrac{M k}{(k-1)!}sum_{j=0}^{k-1}binom{k-1}j
      =frac{Mk}{(k-1)!}2^{k-1}\&=2kMprod_{j=0}^{k-4}frac2{k-1-j}end{align}tag2$$



      Then it is easy to check that for $kge 6$ we have that $frac{|B_k|}{k!}le 16Mfrac{k}{(k-3)^3}$, so by the integral test the series $sum_{k=1}^infty |B_k|/k!$ converges.



      Now using the first approach on the question we can apply the dominated convergence theorem and find that



      $$lim_{ntoinfty}sum_{k=1}^{n+1}frac{B_k}{k!}cdotfrac{n^underline{k-1}}{n^{k-1}}=lim_{ntoinfty}int_{Bbb N_{ge 1}}a_n(k) ,dmathcal H^0=int_{Bbb N_{ge 1}}frac{B_k}{k!},dmathcal H^0=sum_{k=1}^inftyfrac{B_k}{k!}tag3$$



      as desired. However I would like to see, if possible, a more simple solution that doesn't use "advanced" theorems as the dominated convergence theorem.



      P.S.: $dmathcal H^0$ is just the counting measure.





      Essentially the same question appear in a mathematical magazine. In short it says that $n^underline{k-1}in n^{k-1}+O(n^{k-2})$ as $ntoinfty$, thus $frac{n^underline{k-1}}{n^{k-1}}in 1+O(n^{-1})$ and we find that



      $$sum_{k=1}^{n+1}frac{B_k}{k!}cdotfrac{n^underline{k-1}}{n^{k-1}}in sum_{k=1}^{n+1}frac{B_k}{k!}+O(n^{-1})sum_{k=1}^{n+1}frac{B_k}{k!}tag4$$



      Then in the article the author take limits on $(4)$, "giving" the wanted identity. However this procedure seems wrong as its said in the last link. I added this to the answer for the curious reader.






      share|cite|improve this answer

























        up vote
        0
        down vote



        accepted







        up vote
        0
        down vote



        accepted






        I find a solution using the recursion



        $$B_k=-sum_{j=0}^{k-1}binom{k}jfrac{B_j}{k-1+j}tag1$$



        and the dominated convergence theorem. From $(1)$ and setting $M:=sup_{kinBbb N}|B_k|/k!$ (what is finite because I know that $sum_{k=1}^infty B_k/k!$ converges) then we have that



        $$begin{align}left|frac{B_k}{k!}right|&=left|frac1{k!}sum_{j=0}^{k-1}binom{k}jfrac{B_j}{k-j+1}right|=left|sum_{j=0}^{k-1}frac{B_j}{j!}frac1{(k-j+1)!}right|\
        &lefrac{M k}{(k-1)!}sum_{j=0}^{k-1}binom{k-1}j
        =frac{Mk}{(k-1)!}2^{k-1}\&=2kMprod_{j=0}^{k-4}frac2{k-1-j}end{align}tag2$$



        Then it is easy to check that for $kge 6$ we have that $frac{|B_k|}{k!}le 16Mfrac{k}{(k-3)^3}$, so by the integral test the series $sum_{k=1}^infty |B_k|/k!$ converges.



        Now using the first approach on the question we can apply the dominated convergence theorem and find that



        $$lim_{ntoinfty}sum_{k=1}^{n+1}frac{B_k}{k!}cdotfrac{n^underline{k-1}}{n^{k-1}}=lim_{ntoinfty}int_{Bbb N_{ge 1}}a_n(k) ,dmathcal H^0=int_{Bbb N_{ge 1}}frac{B_k}{k!},dmathcal H^0=sum_{k=1}^inftyfrac{B_k}{k!}tag3$$



        as desired. However I would like to see, if possible, a more simple solution that doesn't use "advanced" theorems as the dominated convergence theorem.



        P.S.: $dmathcal H^0$ is just the counting measure.





        Essentially the same question appear in a mathematical magazine. In short it says that $n^underline{k-1}in n^{k-1}+O(n^{k-2})$ as $ntoinfty$, thus $frac{n^underline{k-1}}{n^{k-1}}in 1+O(n^{-1})$ and we find that



        $$sum_{k=1}^{n+1}frac{B_k}{k!}cdotfrac{n^underline{k-1}}{n^{k-1}}in sum_{k=1}^{n+1}frac{B_k}{k!}+O(n^{-1})sum_{k=1}^{n+1}frac{B_k}{k!}tag4$$



        Then in the article the author take limits on $(4)$, "giving" the wanted identity. However this procedure seems wrong as its said in the last link. I added this to the answer for the curious reader.






        share|cite|improve this answer














        I find a solution using the recursion



        $$B_k=-sum_{j=0}^{k-1}binom{k}jfrac{B_j}{k-1+j}tag1$$



        and the dominated convergence theorem. From $(1)$ and setting $M:=sup_{kinBbb N}|B_k|/k!$ (what is finite because I know that $sum_{k=1}^infty B_k/k!$ converges) then we have that



        $$begin{align}left|frac{B_k}{k!}right|&=left|frac1{k!}sum_{j=0}^{k-1}binom{k}jfrac{B_j}{k-j+1}right|=left|sum_{j=0}^{k-1}frac{B_j}{j!}frac1{(k-j+1)!}right|\
        &lefrac{M k}{(k-1)!}sum_{j=0}^{k-1}binom{k-1}j
        =frac{Mk}{(k-1)!}2^{k-1}\&=2kMprod_{j=0}^{k-4}frac2{k-1-j}end{align}tag2$$



        Then it is easy to check that for $kge 6$ we have that $frac{|B_k|}{k!}le 16Mfrac{k}{(k-3)^3}$, so by the integral test the series $sum_{k=1}^infty |B_k|/k!$ converges.



        Now using the first approach on the question we can apply the dominated convergence theorem and find that



        $$lim_{ntoinfty}sum_{k=1}^{n+1}frac{B_k}{k!}cdotfrac{n^underline{k-1}}{n^{k-1}}=lim_{ntoinfty}int_{Bbb N_{ge 1}}a_n(k) ,dmathcal H^0=int_{Bbb N_{ge 1}}frac{B_k}{k!},dmathcal H^0=sum_{k=1}^inftyfrac{B_k}{k!}tag3$$



        as desired. However I would like to see, if possible, a more simple solution that doesn't use "advanced" theorems as the dominated convergence theorem.



        P.S.: $dmathcal H^0$ is just the counting measure.





        Essentially the same question appear in a mathematical magazine. In short it says that $n^underline{k-1}in n^{k-1}+O(n^{k-2})$ as $ntoinfty$, thus $frac{n^underline{k-1}}{n^{k-1}}in 1+O(n^{-1})$ and we find that



        $$sum_{k=1}^{n+1}frac{B_k}{k!}cdotfrac{n^underline{k-1}}{n^{k-1}}in sum_{k=1}^{n+1}frac{B_k}{k!}+O(n^{-1})sum_{k=1}^{n+1}frac{B_k}{k!}tag4$$



        Then in the article the author take limits on $(4)$, "giving" the wanted identity. However this procedure seems wrong as its said in the last link. I added this to the answer for the curious reader.







        share|cite|improve this answer














        share|cite|improve this answer



        share|cite|improve this answer








        edited Nov 19 at 19:55

























        answered Nov 19 at 14:17









        Masacroso

        12.3k41746




        12.3k41746






























            draft saved

            draft discarded




















































            Thanks for contributing an answer to Mathematics Stack Exchange!


            • Please be sure to answer the question. Provide details and share your research!

            But avoid



            • Asking for help, clarification, or responding to other answers.

            • Making statements based on opinion; back them up with references or personal experience.


            Use MathJax to format equations. MathJax reference.


            To learn more, see our tips on writing great answers.





            Some of your past answers have not been well-received, and you're in danger of being blocked from answering.


            Please pay close attention to the following guidance:


            • Please be sure to answer the question. Provide details and share your research!

            But avoid



            • Asking for help, clarification, or responding to other answers.

            • Making statements based on opinion; back them up with references or personal experience.


            To learn more, see our tips on writing great answers.




            draft saved


            draft discarded














            StackExchange.ready(
            function () {
            StackExchange.openid.initPostLogin('.new-post-login', 'https%3a%2f%2fmath.stackexchange.com%2fquestions%2f3004849%2fshow-that-lim-n-sum-k-1n-fracb-kk-fracn-underlinek-1nk-1%23new-answer', 'question_page');
            }
            );

            Post as a guest















            Required, but never shown





















































            Required, but never shown














            Required, but never shown












            Required, but never shown







            Required, but never shown

































            Required, but never shown














            Required, but never shown












            Required, but never shown







            Required, but never shown







            Popular posts from this blog

            Quarter-circle Tiles

            build a pushdown automaton that recognizes the reverse language of a given pushdown automaton?

            Mont Emei